Why don't we allow a linear programming problem to have strictly '' constraints?












1












$begingroup$


I am new to linear programming and I have been asked this question
"Why don't we allow a linear programming problem to have strictly '<' or '>' constraints?"
But unable to answer it.



Kindly provide me an explanation on this.










share|cite|improve this question









$endgroup$

















    1












    $begingroup$


    I am new to linear programming and I have been asked this question
    "Why don't we allow a linear programming problem to have strictly '<' or '>' constraints?"
    But unable to answer it.



    Kindly provide me an explanation on this.










    share|cite|improve this question









    $endgroup$















      1












      1








      1





      $begingroup$


      I am new to linear programming and I have been asked this question
      "Why don't we allow a linear programming problem to have strictly '<' or '>' constraints?"
      But unable to answer it.



      Kindly provide me an explanation on this.










      share|cite|improve this question









      $endgroup$




      I am new to linear programming and I have been asked this question
      "Why don't we allow a linear programming problem to have strictly '<' or '>' constraints?"
      But unable to answer it.



      Kindly provide me an explanation on this.







      linear-programming






      share|cite|improve this question













      share|cite|improve this question











      share|cite|improve this question




      share|cite|improve this question










      asked Oct 12 '16 at 17:59









      ArpitgtArpitgt

      61




      61






















          2 Answers
          2






          active

          oldest

          votes


















          3












          $begingroup$

          Consider the linear program on $mathbb R$ consisting of one constraint: $x < 1$, with the function to be optimized being $f(x) = x$. What's the optimum? At what point is it achieved?



          Answer: There's no optimum. Normally, it'd be at $x = 1$, but that just barely fails to meet the constraint. But for any $x$ less than $1$, there's a better solution, namely $(1+x)/2$.






          share|cite|improve this answer









          $endgroup$





















            0












            $begingroup$

            Because of The Divisibility Assumption.



            The Divisibility Assumption requires that each decision variable is allowed to assume fractional values. For example, the Divisibility Assumption implies that it is acceptable to produce $1.5$ or $1.63$ of a product or service.






            share|cite|improve this answer











            $endgroup$













              Your Answer





              StackExchange.ifUsing("editor", function () {
              return StackExchange.using("mathjaxEditing", function () {
              StackExchange.MarkdownEditor.creationCallbacks.add(function (editor, postfix) {
              StackExchange.mathjaxEditing.prepareWmdForMathJax(editor, postfix, [["$", "$"], ["\\(","\\)"]]);
              });
              });
              }, "mathjax-editing");

              StackExchange.ready(function() {
              var channelOptions = {
              tags: "".split(" "),
              id: "69"
              };
              initTagRenderer("".split(" "), "".split(" "), channelOptions);

              StackExchange.using("externalEditor", function() {
              // Have to fire editor after snippets, if snippets enabled
              if (StackExchange.settings.snippets.snippetsEnabled) {
              StackExchange.using("snippets", function() {
              createEditor();
              });
              }
              else {
              createEditor();
              }
              });

              function createEditor() {
              StackExchange.prepareEditor({
              heartbeatType: 'answer',
              autoActivateHeartbeat: false,
              convertImagesToLinks: true,
              noModals: true,
              showLowRepImageUploadWarning: true,
              reputationToPostImages: 10,
              bindNavPrevention: true,
              postfix: "",
              imageUploader: {
              brandingHtml: "Powered by u003ca class="icon-imgur-white" href="https://imgur.com/"u003eu003c/au003e",
              contentPolicyHtml: "User contributions licensed under u003ca href="https://creativecommons.org/licenses/by-sa/3.0/"u003ecc by-sa 3.0 with attribution requiredu003c/au003e u003ca href="https://stackoverflow.com/legal/content-policy"u003e(content policy)u003c/au003e",
              allowUrls: true
              },
              noCode: true, onDemand: true,
              discardSelector: ".discard-answer"
              ,immediatelyShowMarkdownHelp:true
              });


              }
              });














              draft saved

              draft discarded


















              StackExchange.ready(
              function () {
              StackExchange.openid.initPostLogin('.new-post-login', 'https%3a%2f%2fmath.stackexchange.com%2fquestions%2f1965637%2fwhy-dont-we-allow-a-linear-programming-problem-to-have-strictly-or-cons%23new-answer', 'question_page');
              }
              );

              Post as a guest















              Required, but never shown

























              2 Answers
              2






              active

              oldest

              votes








              2 Answers
              2






              active

              oldest

              votes









              active

              oldest

              votes






              active

              oldest

              votes









              3












              $begingroup$

              Consider the linear program on $mathbb R$ consisting of one constraint: $x < 1$, with the function to be optimized being $f(x) = x$. What's the optimum? At what point is it achieved?



              Answer: There's no optimum. Normally, it'd be at $x = 1$, but that just barely fails to meet the constraint. But for any $x$ less than $1$, there's a better solution, namely $(1+x)/2$.






              share|cite|improve this answer









              $endgroup$


















                3












                $begingroup$

                Consider the linear program on $mathbb R$ consisting of one constraint: $x < 1$, with the function to be optimized being $f(x) = x$. What's the optimum? At what point is it achieved?



                Answer: There's no optimum. Normally, it'd be at $x = 1$, but that just barely fails to meet the constraint. But for any $x$ less than $1$, there's a better solution, namely $(1+x)/2$.






                share|cite|improve this answer









                $endgroup$
















                  3












                  3








                  3





                  $begingroup$

                  Consider the linear program on $mathbb R$ consisting of one constraint: $x < 1$, with the function to be optimized being $f(x) = x$. What's the optimum? At what point is it achieved?



                  Answer: There's no optimum. Normally, it'd be at $x = 1$, but that just barely fails to meet the constraint. But for any $x$ less than $1$, there's a better solution, namely $(1+x)/2$.






                  share|cite|improve this answer









                  $endgroup$



                  Consider the linear program on $mathbb R$ consisting of one constraint: $x < 1$, with the function to be optimized being $f(x) = x$. What's the optimum? At what point is it achieved?



                  Answer: There's no optimum. Normally, it'd be at $x = 1$, but that just barely fails to meet the constraint. But for any $x$ less than $1$, there's a better solution, namely $(1+x)/2$.







                  share|cite|improve this answer












                  share|cite|improve this answer



                  share|cite|improve this answer










                  answered Oct 12 '16 at 18:05









                  John HughesJohn Hughes

                  63.3k24090




                  63.3k24090























                      0












                      $begingroup$

                      Because of The Divisibility Assumption.



                      The Divisibility Assumption requires that each decision variable is allowed to assume fractional values. For example, the Divisibility Assumption implies that it is acceptable to produce $1.5$ or $1.63$ of a product or service.






                      share|cite|improve this answer











                      $endgroup$


















                        0












                        $begingroup$

                        Because of The Divisibility Assumption.



                        The Divisibility Assumption requires that each decision variable is allowed to assume fractional values. For example, the Divisibility Assumption implies that it is acceptable to produce $1.5$ or $1.63$ of a product or service.






                        share|cite|improve this answer











                        $endgroup$
















                          0












                          0








                          0





                          $begingroup$

                          Because of The Divisibility Assumption.



                          The Divisibility Assumption requires that each decision variable is allowed to assume fractional values. For example, the Divisibility Assumption implies that it is acceptable to produce $1.5$ or $1.63$ of a product or service.






                          share|cite|improve this answer











                          $endgroup$



                          Because of The Divisibility Assumption.



                          The Divisibility Assumption requires that each decision variable is allowed to assume fractional values. For example, the Divisibility Assumption implies that it is acceptable to produce $1.5$ or $1.63$ of a product or service.







                          share|cite|improve this answer














                          share|cite|improve this answer



                          share|cite|improve this answer








                          edited Dec 3 '18 at 18:19









                          dantopa

                          6,46942243




                          6,46942243










                          answered Dec 3 '18 at 18:15









                          Ashley MorganAshley Morgan

                          1




                          1






























                              draft saved

                              draft discarded




















































                              Thanks for contributing an answer to Mathematics Stack Exchange!


                              • Please be sure to answer the question. Provide details and share your research!

                              But avoid



                              • Asking for help, clarification, or responding to other answers.

                              • Making statements based on opinion; back them up with references or personal experience.


                              Use MathJax to format equations. MathJax reference.


                              To learn more, see our tips on writing great answers.




                              draft saved


                              draft discarded














                              StackExchange.ready(
                              function () {
                              StackExchange.openid.initPostLogin('.new-post-login', 'https%3a%2f%2fmath.stackexchange.com%2fquestions%2f1965637%2fwhy-dont-we-allow-a-linear-programming-problem-to-have-strictly-or-cons%23new-answer', 'question_page');
                              }
                              );

                              Post as a guest















                              Required, but never shown





















































                              Required, but never shown














                              Required, but never shown












                              Required, but never shown







                              Required, but never shown

































                              Required, but never shown














                              Required, but never shown












                              Required, but never shown







                              Required, but never shown







                              Popular posts from this blog

                              Plaza Victoria

                              In PowerPoint, is there a keyboard shortcut for bulleted / numbered list?

                              How to put 3 figures in Latex with 2 figures side by side and 1 below these side by side images but in...